Как понять неоднозначность разрешения вектора?

Когда мы решаем задачи, где есть маятник, подвешенный на натянутой нерастяжимой струне, и вопрос касается натяжения нити в самой высокой точке качания груза. Ниже приводится традиционный подход к решению проблемы.

введите изображение

Как видите, даже я снял напряжение и м г на соответствующие им компоненты. Путаница у меня была - здесь Т "=" м г потому что θ и м г "=" Т потому что θ . Как мне узнать, какой из них следует учитывать? Потому что они оба имеют одинаковый смысл (по крайней мере, для меня) — их направления идеально совпадают.

осторожно: маятник не находится в статическом равновесии; оно ускоряется.
Буду признателен, если вы дадите мне полный ответ , а не дадите мне подсказки. Я достаточно долго копался в этой проблеме.
Я считаю, что точка зрения @AndrewSteane заключается в том, что ни одно из ваших уравнений не верно, потому что чистая сила, действующая на боб, не равна нулю. Я пишу более полный ответ сейчас.
Ох, ладно. Тогда как правильно поступить? Мне очень жаль, что я вынужден просить вас кормить меня с ложки, но я вроде как новичок во всем этом, и мне трудно обдумать это.
(На самом деле мой комментарий выше не соответствует действительности, т.к. Т "=" м г потому что θ равенство здесь выполняется. Смотрите мой ответ, как только он будет опубликован.)
Хорошо, спасибо. Я жду
@AndrewSteane Я забыл упомянуть, что ситуация, которую я нарисовал, относится к моменту, когда боб находится в самой высокой точке. Кажется, это называется "Динамическое равновесие".
какое программное обеспечение вы использовали для рисования этого FBD?
Я использовал Microsoft OneNote

Ответы (3)

Как указано в комментариях, одно из ваших уравнений неверно, поскольку оно предполагает, что боб не ускоряется вертикально в самой высокой точке. Несмотря на то, что на самом деле это не основная цель вашего вопроса, давайте сначала поработаем с правильными уравнениями, чтобы мы могли избавиться от этого.

В радиальном направлении имеем центростремительное ускорение, поэтому вдоль силы натяжения имеем

Т м г потому что θ "=" м а с "=" м в 2 л
где л длина строки и в это скорость боба. Если боб находится на максимальной высоте, то в "=" 0 и Т "=" м г потому что θ .

По вертикали имеем некоторое ускорение а у такой, что

Т потому что θ м г "=" м а у

На максимальной высоте а у 0 , так м г "=" Т потому что θ здесь не держит. Однако оно будет действительным в некоторой точке между максимальной и минимальной высотами, поскольку вертикальное ускорение должно изменить знак в какой-то момент в течение этого времени.

Теперь давайте перейдем к вашей концептуальной проблеме.

Как мне узнать, какой из них следует учитывать? Потому что они оба имеют одинаковый смысл.

Ты прав! Оба эти уравнения являются действительными. То, что вы хотите использовать, зависит от того, на что вы смотрите. То, что уравнение верно, не означает, что оно полезно. Например, здесь сохраняется энергия, поэтому мы могли бы также иметь правильное уравнение, связывающее максимальную скорость боба с его максимальной высотой над самой низкой точкой (при условии, что он не делает полные петли вокруг точки вращения).

1 2 м в Макс 2 "=" м г у Макс

но если нас не волнуют максимальная скорость или максимальная высота, то это уравнение для нас малопригодно.

Итак, если вас волнует центростремительное ускорение, возможно, используйте это уравнение. Если вы хотите посмотреть на вертикальное движение, возможно, посмотрите на это. Многие уравнения могут быть действительными для аспекта системы; вам нужно узнать, какие уравнения полезны для того, что вы хотите сделать. Обратите внимание и на противопоставление: то, что уравнение бесполезно, не означает, что оно неверно; это то, с чем, как я вижу, сталкиваются новые студенты-физики.

Мне очень жаль, что я немного неправильно сформулировал свой вопрос. Пожалуйста, пройдите последнее редактирование.
Спасибо хоть. Ваш ответ все же прояснил много неправильных представлений
@HarshDarji Готово. Хотя самое последнее редактирование не имеет отношения к сути вашего вопроса.
Я не это хотел сказать. Я предполагаю, что боб находится в самой высокой точке. Вы сказали: как указано в комментариях, ваши уравнения неверны, поскольку предполагается, что боб не ускоряется.
Хотя да... недавнее редактирование не имеет значения. Я думаю, это просто немного конкретизирует ситуацию.
@HarshDarji Боб все еще ускоряется в самой высокой точке. Но тогда я думаю, что одно из ваших уравнений верно. Я отредактирую.
Можете ли вы также уточнить это? Как ускоряется боб, когда он на мгновение находится в покое (в самой высокой точке)?
@HarshDarji Потому что его вертикальная скорость все еще меняется. Вспомните бросок мяча в воздух. В вершине его ускорение все еще г даже если он временно находится в покое.
О, да! Огромное спасибо! Вы буквально спасаете жизни! Я так благодарен за помощь, которую вы мне оказали.

Второй закон Ньютона Ф "=" м а . С точки зрения компонентов вектора это становится

Ф Икс "=" м а Икс Ф у "=" м а у .
В зависимости от того, как мы установили наши координаты, у нас может быть один или оба из а Икс или а у неисчезающий. Но мы можем использовать нашу свободу выбора осей координат, как бы мы ни хотели упростить свою жизнь. В частности, если мы выберем наши координатные оси так, чтобы а точки вдоль одной из них, то другая компонента а равен нулю. Это приводит к исчезновению одного из членов, что упрощает алгебру.

В данном случае мы можем выбрать наши оси так, чтобы Икс -ось указывает вдоль дуги, которую будет описывать боб, а у -ось указывает вдоль строки. Поскольку мы находимся в высшей точке качелей, центростремительного ускорения нет; поэтому ускорение а будет в Икс -только направление, с а у "=" 0 и а Икс "=" а . Тогда уравнения становятся

м г грех θ "=" м а Т м г потому что θ "=" 0.
Эта система уравнений особенно легко решается для неизвестных Т и а : Мы будем иметь Т "=" м г потому что θ и а "=" г грех θ .

Но важно отметить, что выбор разных осей сам по себе не является неправильным ; это только усложняет алгебру. Например, предположим, что вместо этого вы выбрали горизонтальную и вертикальную оси. В этом случае у вас будет а Икс "=" а потому что θ & а у "=" а грех θ , и второй закон Ньютона будет (в этих компонентах)

Т грех θ "=" м а потому что θ Т потому что θ м г "=" м а грех θ .
Эту систему уравнений сложнее решить для а и Т , но можно показать, что решение для а и Т точно так же, как мы получили выше. (Попробуй это!)

Мистер Зайферт, если этот маятник не находится в верхней точке своего качания, "Т" также обеспечивает центростремительную силу.
@DavidWhite: Да, я забыл упомянуть об этом предположении в первоначальной версии моего ответа. Это отредактировано, чтобы упомянуть это предположение сейчас.
@DavidWhite похоже, что положение боба вызывает много проблем. Позвольте мне сделать это немного более конкретным
@DavidWhite Вы можете просмотреть мою последнюю правку
@HarshDarji, положение боба не вызывало путаницы. Что неоднозначно, так это тангенциальная скорость боба для данного изображения.

mg sin(θ) создает крутящий момент, вызывающий угловое ускорение. T - mg cos(θ) не равен нулю. Он должен обеспечивать центростремительное ускорение. mg – T cos(θ) дает нисходящую составляющую ускорения.

Пожалуйста, просмотрите мое последнее редактирование